Proof for Inverse Roots Polynomial

If you don't have time or don't feel like readings lot (or what ever reason you may have) there is an abridged version at the bottom that is much easier to read.

In this note, I will prove that for all polynomials f(x)=anxn+an1xn1+...+a0f(x)=a_n x^n+a_{n-1}x^{n-1}+...+a_0, with an and a00a_n ~ and ~ a_0 \neq 0 and x=0 is not a root, that after being divided by GCD of the coefficients (gg) or multiplied by a factor of kk such that the new coefficients are both integral and co-prime, the function with the smallest possible integral inverseinverse roots has the form f(x)=p(a0xn+a1xn1...+an)f'(x)=p(a_0x^n+a_1x^{n-1}...+a_n) where p is a constant and p=1p=1 if and only if f(x)f(x) has all group wise co-prime integer coefficients.

(In the short, I'm proving that the coefficients are reversed if the roots are inverted (AKA: r1r_1 becomes 1r1\dfrac{1}{r_1})


By Vieta's, we have an1an=r1+r2+r3+rn-\dfrac{a_{n-1}}{a_n}=r_1+r_2+r_3 \dots+ r_n for f(x)f(x)

Now, assume that we didn't know that f(x)f'(x) had the reverse coefficients of f(x)f(x)

Thus f(x)=bnxn+bn1xn1...+b0f'(x)=b_nx^n+b_{n-1}x^{n-1}...+b_0

However, we know that the roots of f(x)f'(x) are 1ri\dfrac{1}{r_i} where 0in0\leq i \leq n.

For f(x)f'(x), we have bn1bn=1r1+1r2+1r3+1rn-\dfrac{b_{n-1}}{b_n}=\dfrac{1}{r_1}+\dfrac{1}{r_2}+\dfrac{1}{r_3} \dots+ \dfrac{1}{r_n}

If we multiply both the top and bottom of every fraction on the R.H.S. by every root other than the one in its denominator, we will have (a1an)(a0an)a1a0-\dfrac{\left(\dfrac{a_1}{a_n}\right)}{\left(\dfrac{a_0}{a_n}\right)}\Rightarrow -\dfrac{a_1}{a_0} (This is too ugly to show in "proper" math format here. So I'll show a simpler and proper version below)

a1a0=bn1bn\therefore -\dfrac{a_1}{a_0}=-\dfrac{b_{n-1}}{b_n}

It can be easily observed that bn=p(a0)b_n=p(a_0) where p is a constant and bn1=p(a1)b_{n-1}=p(a_1).

This process can be repeated to show that ai+1ai=bn(i+1)bn(i)\dfrac{a_{i+1}}{a_{i}}=\dfrac{b_{n-(i+1)}}{b_{n-(i)}} where 0i<n0\leq i < n (NOTE: this is the one time where i<ni<n rather than ini \leq n because there is no b1b_{-1} nor an+1a_{n+1}

We can see that the ratio's of the aia_i is equal to the ratio's of the bib_i in reverse order. Going back to our other equation a1a0=bn1bn\dfrac{a_1}{a_0}=\dfrac{b_{n-1}}{b_n}. Assuming that f(x)f(x) has all group wisegroup ~ wise co-prime coefficients, a1a0\dfrac{a_1}{a_0} could be reducible by a factor of h. However, this leads to two contradictions: 1. every other ratio would have to be reduced by h or f(x)f(x) would be changed. 2. If every other coefficient was reduced then some would become non-integers if they were all group wise co-prime (if they weren't then this would contradict a condition earlier).

Also since we're looking for an f(x)f'(x) with the smallest possible co-prime integer coefficients, bn1bn\dfrac{b_{n-1}}{b_n} should be irreducible. These statements imply that bn1=a1b_{n-1}=a_1 and bn=a0b_n=a_0.

Once again, we can use this logic to prove that ai=bnia_i=b_{n-i} where 0in0\leq i \leq n.

Thus we have proved that for integral coefficients of f(x)f(x), f(x)=a0xn+a1xn1...+anf'(x)=a_0x^n+a_1x^{n-1}...+a_n

If f(x)f(x) doesn't have integer coefficients, then we can multiply by a factor of pp such that the coefficients become integral, thus our final result is f(x)=p(a0xn+a1xn1...+an)f'(x)=p(a_0x^n+a_1x^{n-1}...+a_n).


Proper Version for omitted part

bn1bn=1r1+1r2+1r3+1rn=i=1n((i=1n(ri))ri)1(i=1n(ri))-\dfrac{b_{n-1}}{b_n}=\dfrac{1}{r_1}+\dfrac{1}{r_2}+\dfrac{1}{r_3} \dots+ \dfrac{1}{r_n}=\displaystyle \sum_{i=1}^{n} \left(\dfrac{\left( \displaystyle \sum_{i=1}^{n} (r_i) \right)}{r_i}\right)\cdot \dfrac{1}{\left( \displaystyle \sum_{i=1}^{n} (r_i) \right)}

i=1n((a0an)ri)1(a0an)=i=1n(1ana0ri)ana0\Rightarrow \displaystyle \sum_{i=1}^{n} \left(\dfrac{\left(\dfrac{a_0}{a_n}\right)}{r_i}\right)\cdot\dfrac{1}{\left(\dfrac{a_0}{a_n}\right)}= \displaystyle \sum_{i=1}^{n} \left(\dfrac{1}{a_n}\cdot \dfrac{a_0}{r_i}\right)\cdot\dfrac{a_n}{a_0}

(1an(a1))ana0=a1a0\Rightarrow \left(\dfrac{1}{a_n}\cdot (a_1)\right)\cdot\dfrac{a_n}{a_0}=\dfrac{a_1}{a_0}


Pretty version for those like myself who hate looking at ugly summations:

Polynomial ax3+bx2+cx+dax^3+bx^2+cx+d has roots p,q,r.

ba=p+q+r-\dfrac{b}{a}=p+q+r

ca=pq+qr+rp\dfrac{c}{a}=pq+qr+rp

da=pqr-\dfrac{d}{a}=pqr

Polynomial hx3+gx2+jx+khx^3+gx^2+jx+k has roots 1p,1q,1r\dfrac{1}{p}, \dfrac{1}{q}, \dfrac{1}{r}

-\dfrac{g}{h}=\dfrac{1}{p}+\dfrac{1}{q}+\dfrac{1}{r}=\dfrac{(pq)}{(pq)r}+\dfrac{(rp)}{(rp)q}+\dfrac{(pq)}{(pq)r}=\dfrac{pq+qr+rp}{pqr}=\Rightarrow -\dfrac{\left(\dfrac{c}{a}\right)}{\left(\dfrac{d}{a}\right)}=\boxed{\color \pink{-\dfrac{c}{d}}}

\dfrac{j}{h}=\dfrac{1}{pq}+\dfrac{1}{qr}+\dfrac{1}{rp}=\dfrac{(r)}{(r)pq}+\dfrac{p}{(p)qr}+\dfrac{(q)}{(q)rp}=\dfrac{p+q+r}{pqr}=\dfrac{\left(\dfrac{b}{a}\right)}{\left(\dfrac{d}{a}\right)}=\boxed{\color \green{\dfrac{b}{d}}}

-\dfrac{k}{h}=\dfrac{1}{pqr}=\dfrac{1}{\left(-\dfrac{d}{a}\right)}=\boxed{\color \red{-\dfrac{a}{d}}}

Assuming a monic polynomial (we can divide by h because dividing by a constant won't change the roots)

hx3+gx2+jx+khx^3+gx^2+jx+k has equivalent roots to x^3+\dfrac{g}{h}x^2+\dfrac{j}{h}x+\dfrac{k}{h}=x^3+\color \pink{\dfrac{c}{d}}x^2+\color \green{\dfrac{b}{d}}x+\color \red{\dfrac{a}{d}}

Multiplying through to rationalize the denominators by d.

dx3+cx2+bx+a\boxed{dx^3+cx^2+bx+a}

#Algebra #NumberTheory #Roots #Inverse #Easymoney

Note by Trevor Arashiro
6 years, 6 months ago

No vote yet
1 vote

  Easy Math Editor

This discussion board is a place to discuss our Daily Challenges and the math and science related to those challenges. Explanations are more than just a solution — they should explain the steps and thinking strategies that you used to obtain the solution. Comments should further the discussion of math and science.

When posting on Brilliant:

  • Use the emojis to react to an explanation, whether you're congratulating a job well done , or just really confused .
  • Ask specific questions about the challenge or the steps in somebody's explanation. Well-posed questions can add a lot to the discussion, but posting "I don't understand!" doesn't help anyone.
  • Try to contribute something new to the discussion, whether it is an extension, generalization or other idea related to the challenge.
  • Stay on topic — we're all here to learn more about math and science, not to hear about your favorite get-rich-quick scheme or current world events.

MarkdownAppears as
*italics* or _italics_ italics
**bold** or __bold__ bold

- bulleted
- list

  • bulleted
  • list

1. numbered
2. list

  1. numbered
  2. list
Note: you must add a full line of space before and after lists for them to show up correctly
paragraph 1

paragraph 2

paragraph 1

paragraph 2

[example link](https://brilliant.org)example link
> This is a quote
This is a quote
    # I indented these lines
    # 4 spaces, and now they show
    # up as a code block.

    print "hello world"
# I indented these lines
# 4 spaces, and now they show
# up as a code block.

print "hello world"
MathAppears as
Remember to wrap math in \( ... \) or \[ ... \] to ensure proper formatting.
2 \times 3 2×3 2 \times 3
2^{34} 234 2^{34}
a_{i-1} ai1 a_{i-1}
\frac{2}{3} 23 \frac{2}{3}
\sqrt{2} 2 \sqrt{2}
\sum_{i=1}^3 i=13 \sum_{i=1}^3
\sin \theta sinθ \sin \theta
\boxed{123} 123 \boxed{123}

Comments

Why not tranform the equation by the substitution x=1yx=\dfrac{1}{y} Its a 1 line proof after that.

Aneesh Kundu - 6 years, 6 months ago

Log in to reply

That transformation is certainly a much quicker approach. Can you add more details about what you did?

For a problem along the same lines, check out Squaring the roots of a polynomial.

Calvin Lin Staff - 6 years, 6 months ago

I'm not sure exactly what you mean, but by your substitution, since we're solving for roots, isn't x=x=\infty

Trevor Arashiro - 6 years, 6 months ago

Log in to reply

Why x=x=\infty?

Aren't we just inverting the roots?

Aneesh Kundu - 6 years, 6 months ago

Log in to reply

@Aneesh Kundu Because you substitute \x=1y\x=\dfrac{1}{y}. And since we're finding roots y=0

Trevor Arashiro - 6 years, 6 months ago

Log in to reply

@Trevor Arashiro NOOOOOOO!!!

I just used yy to take a different variable. I did not mean the yy in y=f(x)y=f(x)

Aneesh Kundu - 6 years, 6 months ago

Log in to reply

@Aneesh Kundu Ahh, I see. I was confused there.

Trevor Arashiro - 6 years, 6 months ago

Note: You should state that x=0 x = 0 is not a solution, or that equivalently a00 a_0 \neq 0 . This would also justify allowing you to divide by a0 a_0 .

Calvin Lin Staff - 6 years, 6 months ago

Log in to reply

Ahh, thanks, thats a good point.

Trevor Arashiro - 6 years, 6 months ago

Thanks bruh

Loexx Manncch - 6 years, 6 months ago

Log in to reply

Chee Bra.

Trevor Arashiro - 6 years, 6 months ago
×

Problem Loading...

Note Loading...

Set Loading...